Difference between revisions of "2012 AMC 12B Problems/Problem 21"
Mathgeek2006 (talk | contribs) |
|||
Line 7: | Line 7: | ||
<math>\textbf{(D)}\ 20\sqrt{2}+13\sqrt{3} | <math>\textbf{(D)}\ 20\sqrt{2}+13\sqrt{3} | ||
\qquad\textbf{(E)}\ 21\sqrt{6}</math> | \qquad\textbf{(E)}\ 21\sqrt{6}</math> | ||
+ | |||
+ | ==Solution== | ||
+ | |||
+ | Extend <math>AF</math> and <math>YE</math> so that they meet at <math>G</math>. Then <math>\angle FEG=\angle GFE=60^{\circ}</math>, so <math>\angle FGE=60^{\circ}</math> and therefore <math>AB</math> is parallel to <math>YE</math>. Also, since <math>AX</math> is parallel and equal to <math>YZ</math>, we get <math>\angle BAX = \angle ZYE</math>, hence <math>\triangle ABX</math> is congruent to <math>\triangle </math>YEZ<math>. We now get </math>YE=AB=40<math>. | ||
+ | |||
+ | Let </math>a_1=EY=40<math>, </math>a_2=AF<math>, and </math>a_3=EF<math>. | ||
+ | |||
+ | Drop a perpendicular line from </math>A<math> to the line of </math>EF<math> that meets line </math>EF<math> at </math>K<math>, and a perpendicular line from </math>Y<math> to the line of </math>EF<math> that meets </math>EF<math> at </math>L<math>, then </math>\triangle AKZ<math> is congruent to </math>\triangle ZLY<math> since </math>\angle YLZ<math> is complementary to </math>\angle KZA<math>. Then we have the following equations: | ||
+ | |||
+ | <cmath>\frac{\sqrt{3}}{2}a_2 = AK=ZL = ZE+\frac{1}{2} a_1</cmath> | ||
+ | <cmath>\frac{\sqrt{3}}{2}a_1 = YL =ZK = ZF+\frac{1}{2} a_2</cmath> | ||
+ | |||
+ | The sum of these two yields that | ||
+ | |||
+ | <cmath>\frac{\sqrt{3}}{2}(a_1+a_2) = \frac{1}{2} (a_1+a_2) + ZE+ZF = \frac{1}{2} (a_1+a_2) + EF</cmath> | ||
+ | <cmath>\frac{\sqrt{3}-1}{2}(a_1+a_2) = 41(\sqrt{3}-1)</cmath> | ||
+ | <cmath>a_1+a_2=82</cmath> | ||
+ | <cmath>a_2=82-40=42.</cmath> | ||
+ | |||
+ | So, we can now use the law of cosines in </math>\triangle AGY<math>: | ||
+ | |||
+ | <cmath> 2AZ^2 = AY^2 = AG^2 + YG^2 - 2AG\cdot YG \cdot \cos 60^{\circ} = (a_2+a_3)^2 + (a_1+a_3)^2 - (a_2+a_3)(a_1+a_3)</cmath> | ||
+ | <cmath> = (41\sqrt{3}+1)^2 + (41\sqrt{3}-1)^2 - (41\sqrt{3}+1)(41\sqrt{3}-1) = 6 \cdot 41^1 + 2 - 3 \cdot 41^2 + 1 = 3 (\cdot 41^2 + 1) = 3\cdot 1682</cmath> | ||
+ | <cmath> AZ^2 = 3 \cdot 841 = 3 \cdot 29^2</cmath> | ||
+ | |||
+ | Therefore </math>AZ = 29\sqrt{3} ... \framebox{A}$ |
Revision as of 22:15, 4 December 2012
Problem
Square is inscribed in equiangular hexagon with on , on , and on . Suppose that , and . What is the side-length of the square?
Solution
Extend and so that they meet at . Then , so and therefore is parallel to . Also, since is parallel and equal to , we get , hence is congruent to YEZYE=AB=40$.
Let$ (Error compiling LaTeX. Unknown error_msg)a_1=EY=40a_2=AFa_3=EF$.
Drop a perpendicular line from$ (Error compiling LaTeX. Unknown error_msg)AEFEFKYEFEFL\triangle AKZ\triangle ZLY\angle YLZ\angle KZA$. Then we have the following equations:
<cmath>\frac{\sqrt{3}}{2}a_2 = AK=ZL = ZE+\frac{1}{2} a_1</cmath> <cmath>\frac{\sqrt{3}}{2}a_1 = YL =ZK = ZF+\frac{1}{2} a_2</cmath>
The sum of these two yields that
<cmath>\frac{\sqrt{3}}{2}(a_1+a_2) = \frac{1}{2} (a_1+a_2) + ZE+ZF = \frac{1}{2} (a_1+a_2) + EF</cmath> <cmath>\frac{\sqrt{3}-1}{2}(a_1+a_2) = 41(\sqrt{3}-1)</cmath> <cmath>a_1+a_2=82</cmath> <cmath>a_2=82-40=42.</cmath>
So, we can now use the law of cosines in$ (Error compiling LaTeX. Unknown error_msg)\triangle AGY$:
<cmath> 2AZ^2 = AY^2 = AG^2 + YG^2 - 2AG\cdot YG \cdot \cos 60^{\circ} = (a_2+a_3)^2 + (a_1+a_3)^2 - (a_2+a_3)(a_1+a_3)</cmath> <cmath> = (41\sqrt{3}+1)^2 + (41\sqrt{3}-1)^2 - (41\sqrt{3}+1)(41\sqrt{3}-1) = 6 \cdot 41^1 + 2 - 3 \cdot 41^2 + 1 = 3 (\cdot 41^2 + 1) = 3\cdot 1682</cmath> <cmath> AZ^2 = 3 \cdot 841 = 3 \cdot 29^2</cmath>
Therefore$ (Error compiling LaTeX. Unknown error_msg)AZ = 29\sqrt{3} ... \framebox{A}$